(HELP ASAP) ABC Is Isosceles m

(HELP ASAP) ABC Is Isosceles M

Answers

Answer 1

Answer:

m∠A = 55°

-----------------------------

A and C are congruent angles opposite to congruent sides of the isosceles triangle.

m∠A = m∠C3x + 40 = x + 503x - x = 50 - 402x = 10x = 5

Find the measure of ∠A

m∠A = 3×5 + 40 = 15 + 40 = 55
Answer 2

Answer:

The angle A is 55°.

Step-by-step explanation:

We know that,

→ <A = <C

Now the value of x will be,

→ 3x + 40 = x + 50

→ 3x - x = 50 - 40

→ 2x = 10

→ x = 10/2

→ [ x = 5 ]

Then the value of m<A is,

→ A = 3x + 40

→ A = 3(5) + 40

→ A = 15 + 40

→ [ A = 55° ]

Hence, the answer is 55°.


Related Questions

Which triangle congruence postulate would be used to prove the following triangles congruent?
1. SSS
2. HL
3. AAS
4. SAS

Answers

According to the SSS Postulate, two triangles are congruent if three of one triangle's sides are congruent with three of another triangle's sides.

Explain about the Congruence postulate?

A postulate is an assertion that is not backed by any evidence. Axiom is another name for a postulate. For instance, you would believe Pam if she said that all of her siblings were at least five feet one tall since you are aware of her height and that of all of her siblings, who are all taller than her.

When two figures or objects in geometry have the same shapes, sizes, or are mirror images of one another, they are said to be congruent.

Shapes that are identical to one another are said to be congruent. Both the matching sides and the corresponding angles match. We must examine all of the shapes' angles and sides in order to accomplish this. Two shapes that are similar to one another can be stacked perfectly.

To learn more about  Congruence postulate refer to:

https://brainly.com/question/29268421

#SPJ13


Two consecutive odd integers have a sum of 40. Find the integers

Answers

Answer:

19 and 21|

Step-by-step explanation:

a + (a+2) = 40

2a + 2 = 40

2a = 40 - 2

2a = 38

a = 38/2

a = 19

a+2 = 21

Check:

19 + 21 = 40

Please Help
Find (f+g)(x)if f(x)=4x2-5x+8 and g(x)=2x2+x-2

Answers

Answer: (f+g)(x) = 6x²- 4x + 6

Step-by-step explanation:

We add f(x) and g(x) together, (4x²-5x+8) + (2x²+x-2)

Add like terms together, (4x²+2x²) + (-5x+x) +(8-2)

We are left with (f+g)(x) = 6x²- 4x + 6

rewrite 11/12 in two different ways

Answers

11
~
12 and another way is to do 12
~
11

A firecracker is fired straight up into the air out of a window of a building. Its height, in feet, is given by
h - 16t² + 96t + 112, where t is the time, in seconds, the fircracker has been in the air.
At some point in its path it reaches a highest point.

Answers

Step-by-step explanation:

I assume our task is to find that point of time, when it reaches the highest point.

and the function is

h(t) = -16t² + 96t + 112

well, a maximum or minimum of a curve (function) is found as a zero solution of the first derivative of the function.

given the nature of the function, the extreme point has to be a maximum (the firecracker goes up and then down again).

h'(t) = -32t + 96

we are looking for the zero :

0 = -32t + 96

32t = 96

t = 3 seconds

so, after 3 seconds, the firecracker reaches its highest point, which is at

-16×3² + 96×3 + 112 = -16×9 + 96×3 + 112 =

= -144 + 288 + 112 = 256 ft

FYI - we know for x = 0 (the starting point) the height of the starting window was 112 ft.

Can someone help me out with this math problem and weather it converges or diverges?

Answers

The given infinite series is divergent.

Given,

[tex]\sum \limits^\infty_{n=1 }{(-1)^{n+1}\frac{3n^2+n+1}{2n^2-1}}[/tex]

it is in the form,

[tex]\sum \limits^\infty_{n=1 }{(-1)^{n+1}\ a_n[/tex]

If the series is convergent then it satisfies,

i)[tex]b_{n+1}\ \leq\ b_n\ for\ all\ n[/tex]

ii)[tex]\lim_{n \to \infty} a_n =0[/tex]

if any of the above condition dissatisfies then the series is divergent

First check the 2nd condition,

[tex]\lim_{n \to \infty}\frac{3n^2+n+1}{2n^2-1}}\\\\= \lim_{n \to \infty}\frac{n^2(3+\frac{1}{n}+\frac{1}{n^2})}{n^2(2-\frac{1}{n^2})}}\\\\=\frac{3+0+0}{2-0}\\\\=\frac{3}{2}[/tex]

Here, [tex]\lim_{n \to \infty} a_n \neq 0[/tex]

Thus, the given series is divergent.

To learn more about infinite series refer here

https://brainly.com/question/15415793

#SPJ1

A computer routine selects one of the integers 1, 2, 3, 4, 5 at random and replicates the process a total of 100 times. Let S denote the sum of the 100 numbers selected. Calculate the approximate probability that S assumes a value between 280 and 320 inclusive.

Answers

Probability that S assumes a value between 280 and 320 inclusive is 0.853.

S = Σ[tex]X_{i}[/tex] where  [tex]X_{i}[/tex] has a uniform distribution on 1, 2, 3, 4, 5, with mean 3 and variance (25 – 1)/12 = 2 (result known, or calculated via E[[tex]X^{2}[/tex]] = 11, or from book of formulae, p10, with a = 1, b = 5, h = 1).

So S ~ N(300, 200) approximately.

P(280 ≤ S ≤ 320) = [tex]P(\frac{279.5-300}{\sqrt{200} } < Z < \frac{320.5-300}{\sqrt{200} })[/tex]

                           = [tex]P(-1.450 < Z < 1.450)[/tex]

                           = 0.853.

Hence, probability is 0.853.

To learn more about probability here:

https://brainly.com/question/11234923

#SPJ1

A road is 8/9 of a mile long. A crew needs to repave 2/5 of the road. How long is the section that needs to be repaved?

Answers

10/14 because 8+2=11 and 9+5=14

find the amount on $ 6,250 at 8%p.a compounded annually for 2 years. also, find the compounded intrest

Answers

The Compound Interest is $7,290.00.

What is Compound Interest?

In order to calculate compound interest, multiply the principal of the original loan by the annual interest rate multiplied by the number of compound periods minus one.

Given:

P= 6250

T= 2 years

r = R/100

r = 8/100

r = 0.08 rate per year,

Then solve the equation for A

A = P[tex](1 + r/n)^{nt[/tex]

A = 6,250.00[tex](1 + 0.08/1)^{(1)(2)[/tex]

A = 6,250.00(1 + 0.08)²

A = $7,290.00

Hence, The Compound Interest is $7,290.00.

Learn more about Compound Interest here:

https://brainly.com/question/14295570

#SPJ1

Allison has 2 aquariums. In each aquarium she has 2 families of guppies and 3 tetras. Leigh has 1
16
aquarium with 10 tetras and 3 families of guppies. Allison and Leigh have the same number of
fish and their guppy families each have the same number of members. How many guppies are in
each family?

PLEASE HELP URGENT

Answers

By solving a linear equation, it can be calculated that

There are 4 guppies in  each family

What is a linear equation?

At first it is important to know about equation.

An equation shows the equality between two algebraic expressions by connecting the two algebraic expressions by an equal to sigh

A one degree equation is known as linear equation.

Here a linear equation needs to be solved

Let the number of guppies in each family be x

Allison has 2 aquariums.

In each aquarium she has 2 families of guppies and 3 tetras.

Total number of fishes Allison have = 2(2x + 3) = 4x + 6

Leigh has 1 aquarium with 10 tetras and 3 families of guppies.

Total number of fishes Leigh have  = 3x + 10

By the problem,

4x + 6 = 3x + 10

4x - 3x = 10 - 6

x = 4

There are 4 guppies in each family

To learn more about linear equation, refer to the link-
https://brainly.com/question/2030026

#SPJ1

Please Help
(f-g)(x)if f(x)=/16x4 and g(x)=3x2-3x-5

Answers

The value of the function  (f - g)(x) is x² + 3x + 5

How to solve functions?

A function is defined as a relation between a set of inputs having one output each.

In a simpler terms, a function relates input and output.

Therefore, let's solve the function (f - g)(x)

f(x) =  √16x⁴

g(x) = 3x² - 3x - 5

Therefore,

(f - g)(x) = f(x) - g(x)

(f - g)(x) = √16x⁴ - (3x² - 3x - 5)

(f - g)(x) = √16x⁴ - 3x² + 3x + 5

(f - g)(x) = 4x² - 3x² + 3x + 5

Therefore,

(f - g)(x) = x² + 3x + 5

learn more on function here: https://brainly.com/question/15607023

#SPJ1

4. Angle X in the triangle below is a right angle.
W
15
X
T
h
-25-
2
20
R
9²4625
9+ 25²..
= 20²
400
-625-625
19² √225
15
(a) What is the measure of angle W?
450
(b) What is the height, h, of the triangle?
15.
/2 pts)

Answers

a ) The required measure of x = 90°

b ) The height of the right angled triangle = = [tex]\sqrt{244}[/tex]

Given is a triangle with measure 15, 20 and 25

a ) Since a perpendicular is dropped from above

the two angles of x are equal because the angle is bisected by an angle bisector

So to check whether the angle x is a right angled or not

we can use the pythagoras theorem and check if the LHS = RHS or not

LHS = 15² + 20² = 225 + 400 = 625

and RHS = 25² = 625

Since LHS = RHS thus the angle is 90 degrees

b ) The height of the right triangle ;

by using the Pythagoras theorem again

12.5² + h² = 20²  { side 25 is bisected = 12.5 }

h² = 400 - 156.25

h = [tex]\sqrt{244}[/tex]

To know more about Pythagoras theorem you may visit the link which is mentioned below:

https://brainly.com/question/21926466

#SPJ13

Elena collects data to investigate the relationship between the number of bananas she buys at the store, , and the total cost of the bananas, . Which value for the correlation coefficient is most likely to match a line of best fit of the form for this situation?

Answers

The correlation coefficient that is most likely to match a line of best fit of the form y = mx + b for this situation is of:

0.9.

What is a correlation coefficient?

The correlation coefficient is an index that measures correlation between two variables, assuming numeric values between -1 and 1.

If the correlation coefficient is positive, the relation is positive, that is, they are direct proportional. If the correlation coefficient is negative, they are inverse proportional.

If the absolute value of the correlation coefficient is greater than 0.6, the relationship between the two variables is classified as strong.

In the context of this problem, the variables are listed as follows:

Number of bananas bought.Total price of the bananas.

The total price increases with the number of bananas bought, and there is a strong positive relationship between these two variables, hence the coefficient should be greater than 0.6 and the correct option is:

0.9.

Missing Information

The problem is given by the image shown at the end of the answer.

More can be learned about correlation coefficients at https://brainly.com/question/16355498

#SPJ1

Please help I'm currently very desperate.

Answers

A sequence of transformations maps ∆ABC to ∆A′B′C′. The sequence of transformations that maps ∆ABC onto ∆A′B′C′ is a reflection across the line x = -3 followed by a reflection across the line y = x.

What is the explanation of the above transformation?

Given that both triangles are congruent, we can infer or deduce that there were no dilations or contractions. Hence, since we are moving in a clockwise direction, we can submit that this was not a rotation but a reflection.

Hence the first sequence of reflection was such that:

∆ABC reflects onto ∆A′B′C′ across the line x = -3; followed by
a reflection across the line y = x.

Note that in geometry, reflection refers to the mirror image of an image. The line through which an image reflects is called the line of reflection.

Learn more about transformation:
https://brainly.com/question/11709244
#SPJ1

assuming all conditions for conducting a hypothesis test are met, what are the null and alternative hypothesis?

Answers

The null and alternative hypothesis regarding the test on whether the mean error is of zero is given by the following option:

D. H0: μ = 0 minutes, H1: μ ≠ 0 minutes.

How to identify the null and the alternative hypothesis in this test?

The first step in identifying the hypothesis is to identify the claim tested, given as follows:

The mean prediction error is equals to zero.

At the null hypothesis, it is tested if there is not enough evidence to go against the claim, hence we suppose that the mean is of zero minutes, as follows:

H0: μ = 0 minutes.

At the alternative hypothesis, it is tested if there is enough evidence to reject the null hypothesis, that is, if there is enough evidence to conclude that the mean prediction error is different of zero minutes, hence it is given as follows:

H1: μ ≠ 0 minutes.

Thus option D gives the correct null and alternative hypothesis in the context of this problem.

More can be learned about the test of an hypothesis at brainly.com/question/13873630

#SPJ1

what does d = math help please..................

Answers

The value of d in the expression, 4d/5+3 = -2-d/5, is -5.

According to the question,

We have the following expression:

4d/5+3 = -2-d/5

Now, moving the terms with the same variables on the left hand side will result in the change of the sign from minus to plus:

4d/5+d/5 -3 = -2

5d/5+3 = -2

d+3 = -2

Now, moving 3 from the left hand side to the right hand side will result in the change of the sign from plus to minus:

d = -2-3

We know that two negative integers are added but the sign before the result remains negative.

d = -5

Hence, the value of d is -5.

To know more about value of d here

https://brainly.com/question/11986664

#SPJ1

Andy and Emily each go to a hardware store to buy wire. The table shows the cost y in dollars for x inches of the wire they need. Andy needs 24 feet of the wire. Emily needs 15 yards of the wire. How much will each of them spend for wire?​

Answers

Step-by-step explanation:

Andy and Emily each go to a hardware store to buy wire. The table shows the cost y in dollars for x inches of the wire they need. Andy needs 24 feet of the wire. Emily needs 15 yards of the wire. How much will each of them spend for wire?​

Looking at your chart:

cost/length in inches

6/100 = $0.06 per inch

8.1/135 = $0.06 per inch

9.6/160 = $0.06 per inch

10.8/180 = $0.06 per inch

So the rate stays at a constant rate of $0.06 per inch.

Andy needs 24 feet or 288 inches at a constant rate of $0.06 per inch.

= 288 * $0.06  

= $17.28

________________________________

Emily needs 15 yards or 540 inches at a constant rate of $0.06 per inch.

= 540 * $0.06  

= $32.40

PLEASE HELP
19- 4x + x² can be written in the form
(x + a)² + b, where a and b are numbers.
Work out the values of a and b.

Answers

Step-by-step explanation:

so, let's do the multiplications and squaring. and then compare the result with the target.

(x + a)² + b = x² + 2ax + a² + b

and the original expression is

x² - 4x + 19

x² = x² done

2ax = -4x

2a = -4

a = -2

a² + b = 19

(-2)² + b = 19

4 + b = 19

b = 15

so, the result is

(x - 2)² + 15

Answer: (x-2)^2+15

Step-by-step explanation:

Given x^2-4x+19

=here b=-4 and c=19

B=b/2=-4/2=-2

C=c-B^2=19-(-2)^2=19-4-15

so

(x-B)^2+C

(x-2)^2+15

1,1,2,3,4,5,7,9,
Find the common value

Answers

1, as it appears twice

A child welfare officer asserts that the mean sleep of young babies is 14 hours a day. A random
sample of 64 babies shows that the mean sleep was only 13 hours 30 minutes with standard
deviation of 3 hours. At 5% level of significance, test the assertion that the mean sleep of babies is
less than 14 hours a day

Answers

There is not enough evidence to conclude that the mean is less than 14 hours, as the p-value of the test is greater than 0.05, using the t-distribution.

What are the hypothesis tested?

At the null hypothesis, it is tested if the mean is not less than 14 hours, hence:

[tex]H_0: \mu \geq 14[/tex]

At the alternative hypothesis, it is tested if there is enough evidence that the mean is less than 14 hours, hence:

[tex]H_1: \mu < 14[/tex]

What is the test statistic?

The test statistic is given by the equation presented as follows:

[tex]t = \frac{\overline{x} - \mu}{\frac{s}{\sqrt{n}}}[/tex]

In which the parameters are defined as follows:

[tex]\overline{x}[/tex] is the sample mean.[tex]\mu[/tex] is the value tested at the null hypothesis.s is the standard deviation of the sample.n is the sample size.

In the context of this problem, the values of these parameters are:

[tex]\overline{x} = 13.5, \mu = 14, s = 3, n = 64[/tex]

Hence the test statistic is:

[tex]t = \frac{\overline{x} - \mu}{\frac{s}{\sqrt{n}}}[/tex]

[tex]t = \frac{13.5 - 14}{\frac{3}{\sqrt{64}}}[/tex]

t = -1.33.

P-value and conclusion

Using a z-distribution calculator, with a left-tailed test, as we are testing if the mean is less than a value, with t = -1.33 and 64 - 1 = 63 df, the p-value of the test is of 0.094.

Since the p-value of the test is greater than the significance level of 0.05, there is not enough evidence to conclude that the mean is less than 14 hours.

More can be learned about the t-distribution at https://brainly.com/question/13873630

#SPJ1

The midpoint of AB is M (5, 1). If the coordinates of A are (3,6), what are
the coordinates of B?

Answers

I think your answer would be (-6,7)

Select all of the following that are ordered pairs of the given function.
h(x) = 3x²
(1, 3)
(-1, -3)
(-1,3)
(1, -3)

Answers

The ordered pairs of the function, h(x) = 3x², are (1,3) and (-1,3).

According to the question,

We have the following information:

h(x) = 3x²

Now, we will look at the options to know the correct option of ordered pairs.

Now, in the options, there are only two values of x (1 and -1). So, we will solve the given function using these two values and we will find the value of h(x).

h(x) = 3x²

h(1) = 3*1*1

h(1) = 3

So, the first ordered pair is (1,3).

When x = -1:

h(x) = 3x²

h(-1) = 3*-1*-1

h(-1) = 3

Now, the second ordered pair of the function is (-1,3).

Hence, the correct options are A and C.

To know more about ordered pairs here

https://brainly.com/question/28931422

#SPJ1

Y is directly promotional to x^2
y = 300 x = 10
Write an equation for y in terms of x

Answers

The proportional equation between y and x^2 is:

y = 3*x^2

How to write the equation?

We know that y is directly proportional to x^2, then we can write:

y = k*x^2

Where k is the constant of proportionality.

We also know that when x = 10, the value of y is 300, replacing that in the above equation we get:

300 = k*(10)^2

Now we can solve this for k:

300 = k*100

300/100 = k

3 = k

Then the equation is:

y = 3*x^2

Learn more about proportional relations:

https://brainly.com/question/1835116

#SPJ1

Quandale Jiga has $2.5 the sussy battle pass cost $0.75 how many battle passes can he buy?

Answers

He can get a total of 3 battle passes which $2.5

Find the Average Rate of Change (AROC) of y = 4(1/2)^x over the interval [-2, 0].
a) 6
b) -6
c) 4
d) -4

Answers

Answer:

b) -6

------------------------

Find the value of the function at the endpoints of the given interval

x = - 2 ⇒ y = 4(1/2)⁻² = 4(2)² = 4(4) = 16x = 0  ⇒ y = 4(1/2)⁰ = 4(1) = 4

Average rate of change is

Change in y / change in x =(4 - 16)/(0 - (-2)) = - 12 / 2 = - 6

Correct choice is B.

Answer:

b)  -6

Step-by-step explanation:

The average rate of change of function f(x) over the interval a ≤ x ≤ b is given by:

[tex]\dfrac{f(b)-f(a)}{b-a}[/tex]

Given the interval is [-2, 0]:

a = -2b = 0

Substitute the endpoints of the interval into the function and solve:

[tex]\begin{aligned}x=-2 \implies f(-2)&=4\left(\dfrac{1}{2}\right)^{-2}\\& = 4(4)\\&=16\end{aligned}[/tex]

[tex]\begin{aligned}x=0 \implies f(0)&=4\left(\dfrac{1}{2}\right)^{0}\\& = 4(1)\\&=4\end{aligned}[/tex]

Therefore:

[tex]\begin{aligned}\implies\dfrac{f(b)-f(a)}{b-a}&=\dfrac{f(0)-f(-2)}{0-(-2)}\\\\&=\dfrac{4-16}{0+2}\\\\&=\dfrac{-12}{2}\\\\&=-6\end{aligned}[/tex]

i) Factorise 3x^2-13x-10

Answers

[tex]3x^{2} -13x-10\\=3x^{2} -15x+2x-10\\=(3x^{2} -15x)+(2x-10)\\=3x(x-5)+2(x-5)\\=(x-5)(3x+2)[/tex]

⇒The answer is  (x-5)(3x+2)

Ms. Jones washed
2
7
of her laundry. Her son washed
1
3
of it. Who washed most of the laundry? How much of the laundry still needs to be washed?

Answers

Her son washed the most laundry. 2/7 is a smaller fraction then 1/3.



I can’t help with the next one because I don’t know how much laundry it was in total.

In TY’s math class, 20% of students earned an A on a test. If there were 30 students in the class, how many got the A

Answers

Answer:

Step-by-step explanation

Answer:

6 students

Step-by-step explanation:

[tex]\frac{1}{5} = \frac{x}{30}[/tex]
[tex]30 = 5x[/tex]
[tex]6=x[/tex]

0.1098 round to nearest hundredth

Answers

Rounded it would be 0.11
The answer is 0.11 to

What is the compound interest if $47,000 is invested for 10 years at 8% compounded continuously?

Answers

i think it is 101,469
Other Questions
Rhombus EFGH is rotated 90 counterclockwise about vertex G. How many pairs of parallel sides does the rotated figure have? let x be the exam grade of a random student taking calculus 1 at uiuc. suppose professor smith takes a random sample of 140 students from the calculus 1 class to see how they did on the exam. in his sample, he got a mean of 86 and a standard deviation of 11. approximate the probability that (the sample mean) would be higher than 88.4 or smaller than 84.4? The big boy was a new weapon invented during the civil war How many grams of a 12.5 % sugar solution contain 56.0 g of sugar? within the department of management at rolling hills state university, there are three jobs: department chairperson, administrative assistant, and professor. there is 1 department chairperson, 1 administrative assistant, and 10 professors. how many positions are there? (note: the question is asking about positions, not jobs). How does religion affect you and your community? Present at least two paragraphs Sarah maintains a blog about her soap-making business, and she has hired someone to create a database for this business. She makes bar and liquid soaps in a variety of scents from different types of ingredients. She sells these soaps in certain incremented quantities to customers around the world. Which is one table the database planner will most likely create?a. the number of followers for Sarahs blogb. Sarahs favorite scentsc. the number of hours Sarah researches ingredientsd. customer contact information some flagellated species fail to produce flagella when grown under labortory conditions. what might be the disncetive factorize 6xy-10xy+15x-25 hy does Orwell use allegory in Animal Farm? Select two options.to exemplify the successes of communism in Russiato expose the corruption of the communists in Russiato represent the struggles of the labor class in Russiato glorify life under the control of the tsars in Russiato criticize animal brutality by communists in Russia How can being aware of the techniques (logical fallacies; ethos, pathos, logos) that advertisers, writers, and speakers use to persuade their audiences make you a better critical thinker? Please write at least 2 sentences. when we engage in passive listening, we are group of answer choices following the talk closely and taking notes. not really interested or we are trying to multitask. engaged in the deliberate and conscious process of attending to, understanding, remembering, evaluating, and responding to messages. evaluating the quality of the information. How does Gwendolyn Brooks use graphical elements in "Truth"? What effect does the use of graphical elements have on the reader's experience of the poem? This is English I pressed history on accident A Smart Loan for People with No Credit History Yet1. What is the traditional method of assessing trust to pay a loan? What is the problem withthis method?2. How does Siroya use mobile data to create a financial identity?3. What were Jenipher's options for getting a loan?4. What is one data point that shows a good propensity to pay a loan back?5. How has Jenipher's life improved as a result of the loan using Siroya's technology?Bitcoin. Sweat. Tide. Meet the Future of Branded Currency1. What is Bitcoin and where did it come from?2. What kinds of businesses are accepting Bitcoin?3. What is the definition of money, according to Kemp?4. What are some other forms of currency in existence now?5. Can you think of other examples of currency? A championship weight lifter did 11,000 J of work on a set of barbells weighing 3680 N. How much gravitational potential energy did the barbells have at the maximum height of the lift? (show the steps you followed to solve the problem - ie. the formula, calculations, and final answer) james advertises in the newspaper to sell his used car for $6,000. eric test drove the car and then offered james $5,400 for the car. in this case: Margulis theory of the origin of mitochondria proposes that mitochondria ______. what does kris want to accomplish by administering attitude surveys? how can those surveys help kris address bella's problem? Commercial concentrated nitric acid, HNO3, is 15.8 M. What volume of concentrated HNO3 would be required to prepare 842 mL of 7.77 M HNO3? What provides a visual model for classifying and comparing various social media services?.